1
$\begingroup$

Alfred Tarski, in his paper "Ueber unerreichbare Kardinalzahlen" Fund. Math. vol 30 (1938) pp 68-89 proves the followig theorem of ZFC "If the cardinal of the set Y is equal to the cardinal of the set of the subsets of Y that are not equipotent with Y, then the cardinal of Y is (strongly) inaccessible". The proof of the paper is rather long and involved. Question: Is there another known simpler proof of this theorem ? Gérard Lang

$\endgroup$
5
  • $\begingroup$ Are you sure you quoted the theorem correctly? The continuum hypothesis implies that a set of size $\aleph_1$ has only $\aleph_1$ countable subsets. $\endgroup$ Jan 19, 2011 at 16:34
  • $\begingroup$ I'm confused by the statement of the theorem. There are $(2^{\aleph_0})^{\aleph_0}=2^{\aleph_0}=\aleph_1$ many countable subsets of $\aleph_1$, assuming $CH$, so this would imply that $\omega_1$ is strongly inaccessible. What am I missing? $\endgroup$ Jan 19, 2011 at 16:38
  • $\begingroup$ YES,I am VERY SORRY, I did not think "inaccessible", but "regular !!! I will write the correct question; Gérard lang $\endgroup$ Jan 19, 2011 at 18:58
  • $\begingroup$ There is no need to write another question, it was already answered here. $\endgroup$ Jan 19, 2011 at 19:43
  • $\begingroup$ Is Tarski using the axiom of choice? $\endgroup$ Jan 19, 2011 at 19:45

1 Answer 1

2
$\begingroup$

In modern notation, it says, "if $\kappa$ is a cardinal and $\kappa ^{< \kappa} = \kappa$, then $\kappa$ is strongly inaccessible." This isn't entirely true since the antecedent holds for $\kappa = \omega$ but $\omega$ isn't considered strongly inaccessible, but that's not a big deal. More importantly, under CH the antecedent will hold of $\aleph _1$ but $\aleph _1$ isn't a limit cardinal. So we need to add the assumptions that $\kappa$ is an uncountable limit cardinal. Given that, we can proceed:

So let's assume $\kappa ^{< \kappa} = \kappa$. First we show $\kappa$ is strong limit: $\kappa \leq 2^{< \kappa} \leq \kappa ^{< \kappa} = \kappa$. Next we show $\kappa$ is regular: Suppose not, then $\kappa ^{< \kappa} = \kappa < \kappa ^{ \mathrm{cf} ( \kappa) } \leq \kappa ^{< \kappa}$, contradiction.

$\endgroup$
1
  • $\begingroup$ Thank you very much. This is the right answer for my very question ! Gérard Lang $\endgroup$ Jan 19, 2011 at 21:48

Your Answer

By clicking “Post Your Answer”, you agree to our terms of service and acknowledge you have read our privacy policy.

Not the answer you're looking for? Browse other questions tagged or ask your own question.